LSAT and Law School Admissions Forum

Get expert LSAT preparation and law school admissions advice from PowerScore Test Preparation.

 TigerJin
  • Posts: 31
  • Joined: Sep 28, 2016
|
#30234
Hello. I do not see how B is the missing assumption. If I negate it, it becomes: "Protection of forests and their ecosystems ARE NOT the only legitimate reason for attempting to prevent or control forest fires." The argument still stands if that is the case, does that not strengthen the argument? It gives the guy more reasons to keep forest fires burning then, right?
 Emily Haney-Caron
PowerScore Staff
  • PowerScore Staff
  • Posts: 577
  • Joined: Jan 12, 2012
|
#30248
Hi TigerJin,

This is a Justify the Conclusion question, so you want to make sure you're using the Justify techniques. Here's the stimulus:

Premise: Forest fires are needed for forests to flourish.
Conclusion: Humans should stop trying to control forest fires.

We want an answer choice that connects flourishing forests and controlling forest fires. B does just that - B says that helping forests is the only good reason to try to control forest fires. It connects the premise and conclusion, and allows the conclusion to be drawn.

I hope that helps!
 TigerJin
  • Posts: 31
  • Joined: Sep 28, 2016
|
#30255
I get it now. I must have been reading that answer choice or the stimulus as something else. It is clear now. With B, the guy is saying the , if the only reason to stop forest fires is to protect the forest, then stop putting out the fires, because the fires are in fact helping the forest. I get it. Thanks.

Too, I did not recognize this question as a Justify question my first time through. Thanks.
 avengingangel
  • Posts: 275
  • Joined: Jun 14, 2016
|
#36641
Still confused how B is the correct answer - after reading the conclusion, it seemed to me that I needed something that connected humans' actions with the importance of forest fires burning naturally, on their own. Not connecting flourishing forests with controlling forest fires—why would I want to connect those two things, if human action/interference is the new information that is introduced in the conclusion? Thanks.
 Adam Tyson
PowerScore Staff
  • PowerScore Staff
  • Posts: 5153
  • Joined: Apr 14, 2011
|
#39501
Human action isn't the only new element in the conclusion, angel; so is the idea of leaving all forest fires completely alone. The evidence is that fires have some positive ecological effects and are, in fact, necessary for the health of many forests. To justify leaving all fires alone, to burn out on their own, we need to add that those ecological concerns are the ONLY concerns. There's no concern about loss of life, or damage to structures, or harm to the ecology in surrounding areas, etc. Think of it mathematically:

Premise: Fires are good for forests

Justify: That's all that matters in deciding whether to control fires

Conclusion: Don't do anything to control fires

We sometimes call that the Justify Formula, where the answer, added to the premises, gives us the conclusion.

Is this answer imperfect? Sure, because we didn't get info about ALL forests, just MANY forests. We also didn't get any info about possible harmful ecological effects of fires that might counter some of the benefits, like loss of wildlife and their habitats. Still, this is the LSAT, so we aren't looking for perfect answers; we are looking for the best answer of the bunch. If you have an answer choice that you feel is better, share your thoughts on that and we'll discuss your thinking. If not, then pick the best answer (which is B) and get moving on to the next question quickly and confidently.

Good luck!
 LSAT2018
  • Posts: 242
  • Joined: Jan 10, 2018
|
#49646
So because this is a sufficient assumption, the answer is worded strongly:
Prevent Forest Fire → Protection of Forest
No Protection of Forest → Not Prevent Forest Fire

Protection of the forests is the only reason to prevent fires, but if protection is no longer needed (forest fires are needed to flourish) then forests need not be prevented from forest fires.

Ps. How are the other answers eliminated? Any help would be appreciated! Thank you!!
Last edited by LSAT2018 on Wed Dec 05, 2018 5:16 am, edited 1 time in total.
 student987
  • Posts: 28
  • Joined: Apr 09, 2018
|
#59587
I'm confused about how you would eliminate (A). I was stuck between A and B; I would appreciate any help!
 Robert Carroll
PowerScore Staff
  • PowerScore Staff
  • Posts: 1787
  • Joined: Dec 06, 2013
|
#61160
L and student,

The explanation of why answer choice (B) is correct looks good. It Justifies the conclusion, so that's all that's needed.

As far as the other answers go:

Answer choice (A) tells me that human intervention will lead to a certain result. But nothing about the undesirability of that result has been proven. So why avoid human intervention? No reason is given.

Answer choice (C) tells me to stop fires in certain situations. The stimulus is all about letting fires burn. This certainly does not help.

Answer choice (D) tells me about human views. Nothing in this tells me what to do as a practical matter.

Answer choice (E) is off base. Consider when, given the context of the stimulus, an ecosystem might be threatened by insects or other predators. All I know of such a threat is (possibly) when forest fires don't burn enough to control such threats. So answer choice (E) tells me not to intervene when fires aren't happening. But the stimulus is saying not to intervene to prevent fires. So this is irrelevant.

Robert Carroll

Get the most out of your LSAT Prep Plus subscription.

Analyze and track your performance with our Testing and Analytics Package.